IBDP Maths analysis and approaches Topic: AHL 3.13 Points of intersection HL Paper 1

Question

Consider the points A(1, −1, 4), B (2, − 2, 5) and O(0, 0, 0).

(a)     Calculate the cosine of the angle between \(\overrightarrow {{\text{OA}}} \) and \(\overrightarrow {{\text{AB}}} \).

(b)     Find a vector equation of the line \({L_1}\) which passes through A and B.

The line \({L_2}\) has equation r = 2i + 4j + 7k + t(2i + j + 3k), where \(t \in \mathbb{R}\) .

(c)     Show that the lines \({L_1}\) and \({L_2}\) intersect and find the coordinates of their point of intersection.

(d)     Find the Cartesian equation of the plane which contains both the line \({L_2}\) and the point A.

▶️Answer/Explanation

Markscheme

(a)     Use of \(\cos \theta  = \frac{{\overrightarrow {{\text{OA}}}  \cdot \overrightarrow {{\text{AB}}} }}{{\left| {\overrightarrow {{\text{OA}}} } \right|\left| {\overrightarrow {{\text{AB}}} } \right|}}\)     (M1)

\({\overrightarrow {{\text{AB}}} }\) = ij + k     A1

\(\left| {\overrightarrow {{\text{AB}}} } \right| = \sqrt 3 \) and \(\left| {\overrightarrow {{\text{OA}}} } \right| = 3\sqrt 2 \)     A1

\(\overrightarrow {{\text{OA}}}  \cdot \overrightarrow {{\text{AB}}} = 6\)     A1

substituting gives \(\cos \theta = \frac{2}{{\sqrt 6 }}\,\,\,\,\,\left( { = \frac{{\sqrt 6 }}{3}} \right)\,\,\,\,\,\)or equivalent     M1     N1

[5 marks]

 

(b)     \({L_1}\) : r = \(\overrightarrow {{\text{OA}}} + s\overrightarrow {{\text{AB}}} \,\,\,\,\,\) or equivalent     (M1)

\({L_1}\) : r = ij + 4k + s(ij + k)\(\,\,\,\,\,\)or equivalent     A1

Note: Award (M1)A0 for omitting “ r = ” in the final answer.

 

[2 marks]

 

(c)     Equating components and forming equations involving s and t     (M1)

1 + s = 2 + 2t , −1 − s = 4 + t , 4 + s = 7 + 3t

Having two of the above three equations     A1A1

Attempting to solve for s or t     (M1)

Finding either s = −3 or t = −2     A1

Explicitly showing that these values satisfy the third equation     R1

Point of intersection is (−2, 2, 1)     A1     N1

Note: Position vector is not acceptable for final A1.

 

[7 marks]

 

(d)     METHOD 1

\(r = \left( {\begin{array}{*{20}{c}}
  1 \\
  { – 1} \\
  4
\end{array}} \right) + \lambda \left( {\begin{array}{*{20}{c}}
  2 \\
  1 \\
  3
\end{array}} \right) + \mu \left( {\begin{array}{*{20}{c}}
  { – 3} \\
  3 \\
  { – 3}
\end{array}} \right)\)     (A1)

\(x = 1 + 2\lambda – 3\mu {\text{ , }}y = – 1 + \lambda + 3\mu {\text{ and }}z = 4 + 3\lambda – 3\mu \)     M1A1

Elimination of the parameters     M1

\(x + y = 3\lambda {\text{ so }}4(x + y) = 12\lambda {\text{ and }}y + z = 4\lambda + 3{\text{ so }}3(y + z) = 12\lambda + 9\)

\(3(y + z) = 4(x + y) + 9\)     A1

Cartesian equation of plane is 4x + y − 3z = −9 (or equivalent)     A1     N1

[6 marks] 

METHOD 2

EITHER

The point (2, 4, 7) lies on the plane.

The vector joining (2, 4, 7) and (1, −1, 4) and 2i + j + 3k are parallel to the plane. So they are perpendicular to the normal to the plane.

(ij + 4k) − (2i + 4j + 7k) = −i − 5j − 3k     (A1)

\(n = \left| {\begin{array}{*{20}{c}}
  i&j&k \\
  { – 1}&{ – 5}&{ – 3} \\
  2&1&3
\end{array}} \right|\)     M1

= −12i − 3j + 9k\(\,\,\,\,\,\)or equivalent parallel vector     A1

OR

\({L_1}\) and \({L_2}\) intersect at D(−2, 2, 1)

\(n = \left| {\begin{array}{*{20}{c}}
  i&j&k \\
  2&1&{ – 3} \\
  { – 3}&3&{ – 3}
\end{array}} \right|\)     M1

= −12i − 3j + 9k\(\,\,\,\,\,\)or equivalent parallel vector     A1

THEN

r\( \cdot \)n = (ij + 4k)\( \cdot \)(−12i − 3j + 9k)     M1

= 27     A1

Cartesian equation of plane is 4x + y −3z = −9 (or equivalent)     A1     N1

[6 marks]

Total [20 marks]

Question

Two boats, A and B , move so that at time t hours, their position vectors, in kilometres, are r\(_A\) = (9t)i + (3 – 6t)j and r\(_B\) = (7 – 4t)i + (7t – 6)j .

a.Find the coordinates of the common point of the paths of the two boats.[4]

 

b.Show that the boats do not collide.[2]

 
▶️Answer/Explanation

Markscheme

METHOD 1

\(9{t_A} = 7 – 4{t_B}\) and

\(3 – 6{t_A} = – 6 + 7{t_B}\)     M1A1

solve simultaneously

\({t_A} = \frac{1}{3},{\text{ }}{t_B} = 1\)     A1 

Note: Only need to see one time for the A1.

therefore meet at (3, 1)     A1

[4 marks]

METHOD 2

path of A is a straight line: \(y = – \frac{2}{3}x + 3\)     M1A1 

Note: Award M1 for an attempt at simultaneous equations.

 path of B is a straight line: \(y = – \frac{7}{4}x + \frac{{25}}{4}\)     A1

\( – \frac{2}{3}x + 3 = – \frac{7}{4}x + \frac{{25}}{4}{\text{ }}( \Rightarrow x = 3)\)

so the common point is (3, 1)     A1

[4 marks]

a.

METHOD 1

boats do not collide because the two times \(\left( {{t_A} = \frac{1}{3},{\text{ }}{t_B} = 1} \right)\)     (A1)

are different     R1

[2 marks]

METHOD 2

for boat A, \(9t = 3 \Rightarrow t = \frac{1}{3}\) and for boat B, \(7 – 4t = 3 \Rightarrow t = 1\)

times are different so boats do not collide     R1AG

[2 marks]

 

Question

Consider the triangle \(ABC\). The points \(P\), \(Q\) and \(R\) are the midpoints of the line segments [\(AB\)], [\(BC\)] and [\(AC\)] respectively.

Let \(\overrightarrow {{\text{OA}}}  = {{a}}\), \(\overrightarrow {{\text{OB}}}  = {{b}}\) and \(\overrightarrow {{\text{OC}}}  = {{c}}\).

a.Find \(\overrightarrow {{\text{BR}}} \) in terms of \({{a}}\), \({{b}}\) and \({{c}}\).[2]

b.(i)     Find a vector equation of the line that passes through \(B\) and \(R\) in terms of \({{a}}\), \({{b}}\) and \({{c}}\) and a parameter \(\lambda \).

(ii)     Find a vector equation of the line that passes through \(A\) and \(Q\) in terms of \({{a}}\), \({{b}}\) and \({{c}}\) and a parameter \(\mu \).

(iii)     Hence show that \(\overrightarrow {{\text{OG}}}  = \frac{1}{3}({{a}} + {{b}} + {{c}})\) given that \(G\) is the point where [\(BR\)] and [\(AQ\)] intersect.[9]

 

c.Show that the line segment [\(CP\)] also includes the point \(G\).[3]

 

d.The coordinates of the points \(A\)\(B\) and \(C\) are \((1,{\text{ }}3,{\text{ }}1)\), \((3,{\text{ }}7,{\text{ }} – 5)\) and \((2,{\text{ }}2,{\text{ }}1)\) respectively.

A point \(X\) is such that [\(GX\)] is perpendicular to the plane \(ABC\).

Given that the tetrahedron \(ABCX\) has volume \({\text{12 unit}}{{\text{s}}^{\text{3}}}\), find possible coordinates

of \(X\).[9]

 
▶️Answer/Explanation

Markscheme

\(\overrightarrow {{\text{BR}}}  = \overrightarrow {{\text{BA}}}  + \overrightarrow {{\text{AR}}} \;\;\;\left( { = \overrightarrow {{\text{BA}}}  + \frac{1}{2}\overrightarrow {{\text{AC}}} } \right)\)     (M1)

\( = ({{a}} – {{b}}) + \frac{1}{2}({{c}} – {{a}})\)

\( = \frac{1}{2}{{a}} – {{b}} + \frac{1}{2}{{c}}\)     A1

[2 marks]

a.

(i)     \({{\text{r}}_{{\text{BR}}}} = {{b}} + \lambda \left( {\frac{1}{2}{{a}} – {{b}} + \frac{1}{2}{{c}}} \right)\;\;\;\left( { = \frac{\lambda }{2}{{a}} + (1 – \lambda ){{b}} + \frac{\lambda }{2}{{c}}} \right)\)     A1A1

Note:     Award A1A0 if the \({\text{r}} = \) is omitted in an otherwise correct expression/equation.

Do not penalise such an omission more than once.

(ii)     \(\overrightarrow {{\text{AQ}}}  =  – {{a}} + \frac{1}{2}{{b}} + \frac{1}{2}{{c}}\)     (A1)

\({{\text{r}}_{{\text{AQ}}}} = {{a}} + \mu \left( { – {{a}} + \frac{1}{2}{{b}} + \frac{1}{2}{{c}}} \right)\;\;\;\left( { = (1 – \mu ){{a}} + \frac{\mu }{2}{{b}} + \frac{\mu }{2}{{c}}} \right)\)     A1

Note:     Accept the use of the same parameter in (i) and (ii).

(iii)     when \(\overrightarrow {{\text{AQ}}} \) and \(\overrightarrow {{\text{BP}}} \) intersect we will have \({{\text{r}}_{{\text{BR}}}} = {{\text{r}}_{{\text{AQ}}}}\)     (M1)

Note:     If the same parameters are used for both equations, award at most M1M1A0A0M1.

\(\frac{\lambda }{2}{{a}} + (1 – \lambda ){{b}} + \frac{\lambda }{2}{{c}} = (1 – \mu ){{a}} + \frac{\mu }{2}{{b}} + \frac{\mu }{2}{{c}}\)

attempt to equate the coefficients of the vectors \({{a}}\), \({{b}}\) and \({{c}}\)     M1

\(\left. {\begin{array}{*{20}{c}} {\frac{\lambda }{2} = 1 – \mu } \\ {1 – \lambda  = \frac{\mu }{2}} \\ {\frac{\lambda }{2} = \frac{\mu }{2}} \end{array}} \right\}\)     (A1)

\(\lambda  = \frac{2}{3}\) or \(\mu  = \frac{2}{3}\)     A1

substituting parameters back into one of the equations     M1

\(\overrightarrow {{\text{OG}}}  = \frac{1}{2} \bullet \frac{2}{3}{{a}} + \left( {1 – \frac{2}{3}} \right){{b}} + \frac{1}{2} \bullet \frac{2}{3}{{c}} = \frac{1}{3}({{a}} + {{b}} + {{c}})\)     AG

Note:     Accept solution by verification.

[9 marks]

b.

\(\overrightarrow {{\text{CP}}}  = \frac{1}{2}{{a}} + \frac{1}{2}{{b}} – {{c}}\)     (M1)A1

so we have that \({{\text{r}}_{{\text{CP}}}} = {{c}} + \beta \left( {\frac{1}{2}{{a}} + \frac{1}{2}{{b}} – {{c}}} \right)\) and when \(\beta  = \frac{2}{3}\) the line passes through

the point \(G\) (ie, with position vector \(\frac{1}{3}({{a}} + {{b}} + {{c}})\))     R1

hence [\(AQ\)], [\(BR\)] and [\(CP\)] all intersect in \(G\)     AG

[3 marks]

c.

\(\overrightarrow {{\text{OG}}}  = \frac{1}{3}\left( {\left( {\begin{array}{*{20}{c}} 1 \\ 3 \\ 1 \end{array}} \right) + \left( {\begin{array}{*{20}{c}} 3 \\ 7 \\ { – 5} \end{array}} \right) + \left( {\begin{array}{*{20}{c}} 2 \\ 2 \\ 1 \end{array}} \right)} \right) = \left( {\begin{array}{*{20}{c}} 2 \\ 4 \\ { – 1} \end{array}} \right)\)     A1

Note:     This independent mark for the vector may be awarded wherever the vector is calculated.

\(\overrightarrow {{\text{AB}}}  \times \overrightarrow {{\text{AC}}}  = \left( {\begin{array}{*{20}{c}} 2 \\ 4 \\ { – 6} \end{array}} \right) \times \left( {\begin{array}{*{20}{c}} 1 \\ { – 1} \\ 0 \end{array}} \right) = \left( {\begin{array}{*{20}{c}} { – 6} \\ { – 6} \\ { – 6} \end{array}} \right)\)     M1A1

\(\overrightarrow {{\text{GX}}}  = \alpha \left( {\begin{array}{*{20}{c}} 1 \\ 1 \\ 1 \end{array}} \right)\)     (M1)

volume of Tetrahedron given by \(\frac{1}{3} \times {\text{Area ABC}} \times {\text{GX}}\)

\( = \frac{1}{3}\left( {\frac{1}{2}\left| {\overrightarrow {{\text{AB}}}  \times \overrightarrow {{\text{AC}}} } \right|} \right) \times {\text{GX}} = 12\)     (M1)(A1)

Note:     Accept alternative methods, for example the use of a scalar triple product.

\( = \frac{1}{6}\sqrt {{{( – 6)}^2} + {{( – 6)}^2} + {{( – 6)}^2}}  \times \sqrt {{\alpha ^2} + {\alpha ^2} + {\alpha ^2}}  = 12\)     (A1)

\( = \frac{1}{6}6\sqrt 3 |\alpha |\sqrt 3  = 12\)

\( \Rightarrow |\alpha | = 4\)     A1

Note:     Condone absence of absolute value.

this gives us the position of \(X\) as \(\left( {\begin{array}{*{20}{c}} 2 \\ 4 \\ { – 1} \end{array}} \right) \pm \left( {\begin{array}{*{20}{c}} 4 \\ 4 \\ 4 \end{array}} \right)\)

\({\text{X}}(6,{\text{ }}8,{\text{ }}3)\) or \(( – 2,{\text{ }}0,{\text{ }} – 5)\)     A1

Note:     Award A1 for either result.

[9 marks]

Total [23 marks]

 
Scroll to Top